Difference between revisions of "2005 AMC 10A Problems/Problem 24"
m (→Solution 2) |
m (→Solution 2) |
||
(3 intermediate revisions by the same user not shown) | |||
Line 56: | Line 56: | ||
Looking at pairs of [[divisor]]s of <math>48</math>, we have several possibilities to solve for <math>p_{1}</math> and <math>p_{2}</math>: | Looking at pairs of [[divisor]]s of <math>48</math>, we have several possibilities to solve for <math>p_{1}</math> and <math>p_{2}</math>: | ||
− | (Note: you can skip several cases below by observing that <math>p_1+p_2</math> and <math>p_1-p_{2}</math> must be even, and p_1+p_2 \neq p_1-p_2 | + | (Note: you can skip several cases below by observing that <math>p_1+p_2</math> and <math>p_1-p_{2}</math> must be even, and <math>p_1+p_2 \neq p_1-p_2 \pmod 4 </math>.) |
− | < | + | <math> (p_{2}+p_{1}) = 48 </math> |
− | < | + | <math> (p_{2}-p_{1}) = 1 </math> (impossible) |
− | < | + | <math> p_{1} = \frac{47}{2} </math> |
− | < | + | <math> p_{2} = \frac{49}{2} </math> |
− | < | + | <math> (p_{2}+p_{1}) = 24 </math> |
− | < | + | <math> (p_{2}-p_{1}) = 2 </math> |
− | < | + | <math> p_{1} = 11 </math> |
− | < | + | <math> p_{2} = 13 </math> (Valid!) |
− | |||
− | < | + | <math> (p_{2}+p_{1}) = 16 </math> |
− | < | + | <math> (p_{2}-p_{1}) = 3 </math> (impossible) |
− | < | + | <math> p_{1} = \frac{13}{2} </math> |
− | < | + | <math> p_{2} = \frac{19}{2} </math> |
− | < | + | <math> (p_{2}+p_{1}) = 12 </math> |
− | < | + | <math> (p_{2}-p_{1}) = 4 </math> (impossible) |
− | < | + | <math> p_{1} = 4 </math> |
− | < | + | <math> p_{2} = 8 </math> |
− | < | + | <math> (p_{2}+p_{1}) = 8 </math> |
− | < | + | <math> (p_{2}-p_{1}) = 6 </math> |
− | < | + | <math> p_{1} = 1 </math> (not prime) |
− | < | + | <math> p_{2} = 7 </math> |
− | The only solution < | + | The only solution <math> (p_{1} , p_{2}) </math> where both numbers are primes is <math>(11,13)</math>. |
− | Therefore the number of [[positive integer]]s < | + | Therefore the number of [[positive integer]]s <math>n</math> that satisfy both statements is <math>\boxed{\textbf{(B) }1}.</math> |
==Solution 3== | ==Solution 3== |
Revision as of 00:37, 9 July 2024
Contents
Problem
For each positive integer , let denote the greatest prime factor of . For how many positive integers is it true that both and ?
Solution 1
If , then , where is a prime number.
If , then is a square, but we know that n is .
This means we just have to check for squares of primes, add and look whether the root is a prime number.
We can easily see that the difference between two consecutive square after is greater than or equal to ,
Hence we have to consider only the prime numbers till .
Squaring prime numbers below including we get the following list.
But adding to a number ending with will result in a number ending with , but we know that a perfect square does not end in , so we can eliminate those cases to get the new list.
Adding , we get as the only possible solution. Hence the answer is .
edited by mobius247
Note: Solution 1
Since all primes greater than are odd, we know that the difference between the squares of any two consecutive primes greater than is at least , where p is the smaller of the consecutive primes. For , . This means that the difference between the squares of any two consecutive primes both greater than is greater than , so and can't both be the squares of primes if and . So, we only need to check and .
~apsid
Video Solution
~rudolf1279
Solution 2
If , then , where is a prime number.
If , then , where is a different prime number.
So:
Since : .
Looking at pairs of divisors of , we have several possibilities to solve for and :
(Note: you can skip several cases below by observing that and must be even, and .)
(impossible)
(Valid!)
(impossible)
(impossible)
(not prime)
The only solution where both numbers are primes is .
Therefore the number of positive integers that satisfy both statements is
Solution 3
For the statement to be true, we must have both and be squares of primes. Support we have the number , where is a positive integer. Then the next perfect square, , is greater than . The next perfect square after that will be greater than . In general, the prime will be greater than . However, we must have that . can take on any value between and (if is equal to , we have , where would have to be negative for the difference to be ). However, we can eliminate all the cases where is odd, because we would then have a number of the form , which is odd because can take only integral values. As such, we consider , , and . If , then . Then our squares are and , both of which are squares of primes. If , then . However, isn't prime, so we discard this case. Finally, if , then . Again, isn't prime, so we discard this case as well. Thus, we only have valid case.
~ cxsmi
Video Solution 2
~savannahsolver
See Also
2005 AMC 10A (Problems • Answer Key • Resources) | ||
Preceded by Problem 23 |
Followed by Problem 25 | |
1 • 2 • 3 • 4 • 5 • 6 • 7 • 8 • 9 • 10 • 11 • 12 • 13 • 14 • 15 • 16 • 17 • 18 • 19 • 20 • 21 • 22 • 23 • 24 • 25 | ||
All AMC 10 Problems and Solutions |
The problems on this page are copyrighted by the Mathematical Association of America's American Mathematics Competitions.